Wir können – weil’s so schick ist – auch diese Funktion in unser dreidimensionales Diagramm eintragen:

i-56db48df612feff03123f612ff887096-vacuum2particle.png

Diesen Zustand können wir entsprechend schreiben als |2 k1⟩. Auch das können wir wieder auf das elektrische Feld von geeignet polarisierten Photonen übertragen. Zwei Photonen können wir uns demnach so veranschaulichen:

i-367a914eee84f84be389abedbc48b6f8-photon1-two.png

Hier haben wir jetzt drei Maxima für die Wahrscheinlichkeit: Eins bei Null und jeweils eins im positiven und negativen Bereich.

Anregungen mit mehr als zwei Teilchen (|3 k1⟩, |4 k1⟩ usw.) zeige ich euch jetzt nicht – das Schema ist immer dasselbe: Sucht einfach die passende Wellenfunktion des harmonischen Oszillators und baut sie in das Diagramm ein.

Viel interessanter ist eine andere Frage: Wie funktioniert eigentlich Laser-Licht? Wenn – wie oben erläutert – der Erwartungswert des elektrischen Feldes für eine beliebige Anzahl von Photonen immer verschwindet, wie kann man dann kohärentes Laserlicht erzeugen? Laserlicht hat eine genau definierte Ausrichtung des elektrischen Feldes, also einen (ziemlich genau) bestimmten Feldwert – für Laserlicht ist diese Darstellung

 

i-a11c3953ddf0134742bcef76ad959db8-emWelle-thumb-500x123.jpg

By SuperManu – Self, based on Image:Onde electromagnetique.png, CC BY-SA 3.0, https://commons.wikimedia.org/w/index.php?curid=2107870

eigentlich eine sehr gute Beschreibung. Da Laserlicht aber ja aus Photonen besteht, muss der Zustand unseres Quantenfelds zu dieser Zeichnung passen – eine unscharfe Überlagerung wie bei unserem Photonenbild ist nicht möglich.

Übertragen auf unsere Schreibweise mit φ heißt das, dass im Laserlicht φ(k1) einen bestimmten Wert annimmt und eben keine Verteilung hat, die bei positiven und negativen Werten denselben Wert hat.

Wie soll das gehen?

Falls ihr selbst knobeln wollt, schaut euch noch mal den Text über den harmonischen Oszillator an, da haben wir etwas ganz ähnliches gesehen.

Als Spoiler Space hier ein Bild von Roy Glauber, der sich über dieses Problem Anfang der 60er Jahre Gedanken gemacht hat und für die Lösung (naja, ein bisschen mehr hat er schon noch getan) einen Nobelpreis bekommen hat – Ihr habt jetzt also die einmalige Gelegenheit, beim Bloglesen mal eben schnell eine nobelpreiswürdige Entdeckung zu machen.

i-42d9e810ebfee78be86d20920c3fb23d-Roy_Glauber_Dec_10_2005.jpg

Falls ihr es noch nicht herausbekommen habt, hier das entscheidende Bild:

i-e2c3a620553b891399eaadf51c509e37-213px-Qmoszi_quasiklassischer_zustand.png

Beim harmonischen Oszillator hatten die Zustände mit definierter Energie ebenfalls eine symmetrische Wahrscheinlichkeitsverteilung (die Wellenfunktionen waren zum Teil asymmetrisch, aber für die Wahrscheinlichkeit werden die ja quadriert), und zwar im Ort. Um ein an einem Ort lokalisiertes Wellenpaket zu bekommen, haben wir verschiedene Energiezustände überlagert.

Dasselbe müssen wir jetzt hier auch tun, ihr braucht nur die Übersetzungstabelle von der Quantenmechanik zur QFT zu verwenden: Um eine Wahrscheinlichkeitsverteilung zu bekommen, die bei einem bestimmten Wert von φ(k1) zentriert ist, müssen wir unterschiedliche Zustände überlagern. Hier in der QFT sind das jetzt Zustände mit unterschiedlich vielen Teilchen. Wir müssen also einen Zustand bauen, der eine Überlagerung von Zuständen mit unterschiedlich vielen Teilchen mit Impuls k1 ist:

(Wenn ihr den Nobelpreis wollt, müsst ihr natürlich – unter anderem – auch die Werte für die a’s ausrechnen – ist allerdings auch nicht schrecklich schwer.

So etwa sieht dieser Zustand in der 3D-Darstellung aus:

i-d2afe5bd69400e23354689338ee9f52a-stateDefinedPhase.png

(Anders als bei den anderen Bildern war ich hier etwas schlampig mit der Normierung – die Höhe des Wellenpakets habe ich nicht exakt nachgerechnet, sondern nur geschätzt.)

Was bedeutet das? Es bedeutet anschaulich, dass der Zustand eines Laserstrahls (man nennt das einen kohärenten Zustand) eine Überlagerung aus unterschiedlichen Zuständen mit unterschiedlicher Teilchenzahl ist. Mit anderen Worten: Die Zahl der Photonen in einem Laserstrahl (und damit auch seine Energie) ist nicht wohldefiniert, sondern wir haben eine quantenmechanische Überlagerung. Wenn ihr also zählt, wie viele Photonen in eurem Laserstrahl sind, dann macht ihr dabei den Laserzustand leider kaputt. Wenn ich es richtig sehe, ist so etwas experimentell in dieser Arbeit gemacht worden.

Wie wir gesehen haben, können wir also für unseren Wert k1 beliebige Überlagerungen der Wellenfunktionen des harmonischen Oszillators zusammenbauen, so wie den kohärenten Zustand. Bisher haben wir aber immer nur eine einzige Wellenzahl k1 betrachtet. Auch das ist natürlich nicht unbedingt notwendig. Man könnte also zum Beispiel auch solche Zustände hier bekommen:

1 / 2 / 3 / 4

Kommentare (88)

  1. #1 SCHWAR_A
    18. Juni 2012

    @MartinB:

    Super-Schluß der Serie!

    Wie kann ich mir mit der Überlagerung den Casimir-Effekt vorstellen? Was erzeugt da die vielen passenden sich überlagernden Photonen? Falls tatsächlich durch Teilchen-Antiteilchen, dann dürften die doch nur bestimmte Impuls-Stärken erzeugen und kein gleichmäßiges Spektrum, oder etwa doch?

    Herzliche Grüße.

  2. #2 MartinB
    18. Juni 2012

    @SCHWAR_A
    Naja, Schluss der Serie ist es vermutlich nicht (obwohl ich erstmal ne QFT-Pause brauche). Beim Casimir-Effekt sollte man sich eben keine Teilchen-Antiteilchen vorstellen. Vielmehr ist es so, dass die Wände die erlaubten k-Werte abschneiden und sich dadurch die im Vakuum zwischen den Wänden gespeicherte Energie ändert. Es gibt eine nette Rechnung dazu im Buch von Zee, die ist nicht mal besodners schwer, braucht aber Auf- und Absteigeoperatoren. Falls ich die mal erkläre, erkläre ich dann auch den Casimir-Effekt nochmal im Detail.

  3. #3 SCHWAR_A
    19. Juni 2012

    @MartinB:
    Das Abschneiden erlaubter k-Werte ist klar, aber nicht, woher überhaupt die
    “zwischen den Wänden gespeicherte Energie” herkommt.

    Immerhin geht die Energiedichte ja mit 1/r^4 und das Speicher-Volumen mit r³ — es ist also nicht nur ein bestimmter Energie-Bestand gespeichert, sondern es wird auch bei r-Änderung Energie “frisch” hinzugefügt bzw. entfernt.

    Ich stelle mir vor, daß die beiden Platten selbst genügend Oszillation erzeugen, also Photonen, die sich derart konfigurieren, daß sich stehende Wellen und Halbwellen zwischen den Platten aufbauen.
    Was meinst du dazu?

    Herzliche Grüße.

  4. #4 MartinB
    19. Juni 2012

    @SCHWAR_A
    “es wird auch bei r-Änderung Energie “frisch” hinzugefügt bzw. entfernt.”
    Ja, das ist etwas tüftelig. Die beste Art, das zu berechnen ist, wenn man drei Platten betrachtet, zwei in festem Abstand und eine dazwischen, die man verschieben kann. Sonst handelt man sich Ärger mit dem unendlichen Außenraum ein. Wenn man es so betrachtet, dann sieht man – wenn ich mich recht erinnere – direkt, wie sich die zwischen den Platten jeweils gespeicherte Energie ändert.
    Die Vakuumenergie ist zunächst mal einfach da – wenn ich die Platte aktiv gegen die Casimirkraft verschiebe, bringe ich dadurch Energie ein, wenn die Platte der Casimirkraft folgt, dann entzieht sie letztlich dem Vakuum Energie.
    Die Vorstellung, dass die Platten irgendwelche Oszillationen “erzeugen” halte ich für falsch und sie wird auch durch nichts in den Formeln repräsentiert, soweit ich das sehe.

  5. #5 SCHWAR_A
    19. Juni 2012

    @MartinB:
    “…und sie wird auch durch nichts in den Formeln repräsentiert, soweit ich das sehe.”

    Na, das sehe ich aber ganz anders:

    ΔE = E-Quelle · gravitative Rotverschiebung · Anzahl Andockpunkte auf Quelle / Volumen(Dichte) · Volumen(Kopplung) · Anzahl Wellen zwischen Platten

    ΔE = (½m_1·c²) · 2G·(½m_1)/(c²d) · κ / (½ · 4/3 πd³) · A·d · π^4/90/(2π)

    Ein Partikel der einen Platte gebe Oszillations-Energie ab, wobei diese Photonen eine gravitative Rotverschiebung erleiden, also Energie rückwärts fließt. Es gebe weiterhin κ Punkte auf dem Partikel, die als Endpunkte für Wellen in Frage kommen. Diese ausgesandte Energie besitzt die E-Dichte bzgl. der Halbkugel mit Radius d.
    Der letzte Term ist Σ (1/i^4) für i=[1,∞[, sowie Elimination aller dadurch zuviel gezählten Endpunkte auf der anderen Platte – die liegen ja alle auf einem Kreis. Die Summe aller 1/i^4 ist die Folge der möglichen Einzel-Energiedichten hc/(2πi(2d))/(½ · 4/3 π(id)³).

    Nach diesem Modell wird also die gravitativ zurückfließende Oszillations-Energie aller Partikel die Anziehungskraft zwischen den Platten erzeugen.

    Die Anzahl Andockpunkte für Wellen κ wäre in diesem Modell

    κ = m_pl²/m_1²

    Hawking’s Entropie schwarzer Löcher bezieht sich auf ein Planck-Längen-Quadrat der betrachteten Oberfläche – genauso scheint dieses κ zu sein: rechnen wir den Oszillator m_1 auf seine Wellenlänge um, erhalten wir seine “Größe”, und dessen Oberfläche bezogen auf das Planck-Längen-Quadrat ist κ — quasi die Entropie des Oszillators m_1.

    Dadurch kann die obige Gleichung umgeformt werden zu

    ΔE = G · m_pl²/(2d) / (½ · 4/3 πd³) · A·d · π^4/90/(2π)
    ΔE = ħc/m_pl² · m_pl²/(2d) / (½ · 4/3 πd³) · A·d · π^4/90/(2π)
    ΔE = hc/(2π(2d)) / (½ · 4/3 πd³) · A·d · π^4/90/(2π)
    ΔE = hcπA / (480d³)

    Das erscheint weit hergeholt, zeigt aber, daß es nicht unmöglich ist, daß Oszillationen in den Platten stecken, und daß das doch in den Formeln steckt…

    Zusätzlich zeigt dieses Modell noch, daß es durchaus auch Gravitation mit ART sein kann, die diesen Effekt erzeugt.

    Herzliche Grüße.

  6. #6 MartinB
    19. Juni 2012

    @SCHWAR_A
    Da scheitere ich schon im Ansatz: Was hat die gravitative Rotverschiebung damit zu tun?
    Und warum sollte ein Partikel der Platte Energie abgeben? Woher soll die kommen (ich kann ja zumindest konzeptionell die Platte auf T=0 setzen)?

  7. #7 SCHWAR_A
    19. Juni 2012

    @MartinB:
    “Was hat die gravitative Rotverschiebung damit zu tun?”

    Die sorgt für rückwärts fließende Energie und damit letztlich für eine Anziehung.

    “…die Platte auf T=0 setzen”

    Die Oszillation muß doch nicht zwingend temperaturbedingt sein. Das Modell stützt sich auf Gravitation – die verschwindet nicht bei T=0…

    Herzliche Grüße.

  8. #8 MartinB
    19. Juni 2012

    @SCHWAR_A
    “Die sorgt für rückwärts fließende Energie und damit letztlich für eine Anziehung.”
    Nein. Denn den Casimireffekt kann man berechnen, ohne die Gravitation zu berücksichtigen, er spielt sich rein innerhalb der QED ab.
    Es ergibt also gar keinen Sinn, nach einem Gravitations-Effekt zu suchen.

    “Das Modell stützt sich auf Gravitation – die verschwindet nicht bei T=0…”
    Nein, aber du hast was von oszillierenden Teilchen geschrieben, und bei T=0 oszilliert nichts mehr. “Ein Partikel der einen Platte gebe Oszillations-Energie ab” Was soll denn das für eine Oszillation sein?

  9. #9 SCHWAR_A
    19. Juni 2012

    @MartinB:
    Stimmt, ich schrieb “Oszillations-Energie” – zu dem Zeitpunkt war es mir noch egal, woher die Energie kommen soll – sie war einfach da, wie bei Dir die Vakuum-Energie.

    Wenn man die Rechnungen von unten nach oben liest, sieht man, daß eine gravitative Interpretation möglich ist, und daß man wieder an einen Punkt kommt, an dem man beschreiben muß, woher die Energie eigentlich kommen soll. Es genügt nicht, zu sagen, sie sei einfach nur da – das gilt in beiden Fällen. Im Gravitations-Modell gibt es aber immerhin bereits Oszillatoren (½m_1·c²).

    Die “Vakuum-Energie” ist keine “magische Energiequelle”. Sie entsteht, wie Du ja beschrieben hast, aus Überlagerungen, die sich nicht herauslöschen. Die Quelle der Energie für diese Überlagerungen ist aber noch ungeklärt (zumindest sehe ich sie noch nicht)….

    Herzliche Grüße.

  10. #10 MartinB
    19. Juni 2012

    @SCHWAR_A
    Wie soll eine Gravitations-Interpretation möglich sein? ich kann den Casimir-Effekt direkt aus der QED ableiten – er ist also unabhängig von der Gravitationskonstante, selbst wenn die Null wäre, gäbe es den Casimir-Effekt.
    “Im Gravitations-Modell gibt es aber immerhin bereits Oszillatoren”
    ??
    In der QFT ja auch, genau das zeigt doch dieser Teil der Serie – jede Mode verhält sich exakt wie ein Oszillator.
    “Die “Vakuum-Energie” ist keine “magische Energiequelle””
    Stimmt.

    “Die Quelle der Energie für diese Überlagerungen ist aber noch ungeklärt (zumindest sehe ich sie noch nicht)….”
    Die Quelle ist dieselbe wie die Nullpunktsenergie eines einzelnen harmonischen Oszillators – wenn du willst, kannst du die Unschärfe nehmen. Es ist aber letztlich nicht möglich, dass ein Phänomen, dass innerhalb der QED abgeleitet werden kann, auf die Gravitation zurückführbar ist (allenfalls könnte man in einer Quantengravitation ein genaueres Verständnis bekommen, warum die Nullpunktsenergie nicht direkt auf die Raumkrümmung wirkt.), wie gesagt schon allein deswegen nicht, weil die Gravitationskonstante nicht eingeht. (Und sicherlich auch schon daran zu sehen, dass das Schwerefeld der Erde viel stärker ist als das der Platten, also müsste es das Ergebnis ja drastisch beeinflussen.)

  11. #11 SCHWAR_A
    19. Juni 2012

    @MartinB:
    “Gravitationskonstante, selbst wenn die Null wäre”

    Die Gravitationskonstante kann nicht Null sein:

    G = ħc/m_pl² = c² l_pl/m_pl

    “Unschärfe”

    Energie aus der Unschärfe kann man doch nur für entsprechend kurze Zeit “leihen”: zu wenig, um mehrere Wellenlängen lang zu sein. Und wenn ich sie “nutze”, sprich, durch Kraft auf die Platten entziehe, kann sie auch nicht zurückkehren, die Unschärfe wäre verletzt.

    Anscheinend verstehe ich den Mechanismus der “Energie-Gewinnung” aus Unschärfe noch nicht wirklich…-(

    “Es ist aber letztlich nicht möglich, dass ein Phänomen, dass innerhalb der QED abgeleitet werden kann, auf die Gravitation zurückführbar ist (allenfalls könnte man in einer Quantengravitation ein genaueres Verständnis bekommen, warum die Nullpunktsenergie nicht direkt auf die Raumkrümmung wirkt.)”

    Mir scheint der Casimir-Effekt ein Zugang zur Quantengravitation zu sein…

    Herzliche Grüße.

  12. #12 SCHWAR_A
    19. Juni 2012

    @MartinB:
    “(Und sicherlich auch schon daran zu sehen, dass das Schwerefeld der Erde viel stärker ist als das der Platten, also müsste es das Ergebnis ja drastisch beeinflussen.)”

    Der Effekt beruht nicht auf dieser Wirkung der normalen Gravitation – der wäre viel zu schwach.
    Der Effekt im Modell beruht auf der sehr großen Anzahl κ an kleinen Energie-Rückflüssen durch die Rotverschiebung im Schwerefeld eines jeden Partikels. Dadurch ist er sehr viel stärker als die normale Gravitation, aber sein Kraft-Verhalten ist proportional zu 1/d^4.
    Die Gravitationskonstante beschreibt hier nur die Stärke eines einzelnen Rückflusses.

    Herzliche Grüße.

  13. #13 MartinB
    19. Juni 2012

    @SCHWAR_A
    Ich raff’s mal wieder nicht: Ist das jetzt deine Privat”theorie” oder kommt das irgendwo her? Geht das Argument, dass man den Casimireffekt komplett ohne Gravitation herleiten kann, eigentlich an dir vorbei? Wenn ja, warum?

  14. #14 SCHWAR_A
    22. Juni 2012

    @MartinB:
    “Geht das Argument, dass man den Casimireffekt komplett ohne Gravitation herleiten kann, eigentlich an dir vorbei? ”

    Natürlich nicht!
    Könnte es sein, daß Du hier lieber das Occam’sche Rasiermesser anwendest und daher NUR die übliche QED-Deutung siehst und dabei alles andere gar nicht erst zuläßt?

    Was ich tue, ist, das beobachtete Ergebnis anders umzuformen, um so eine andere physikalische Deutung zu erlangen.

    (1) Der Casimir-Effekt kann komplett ohne Gravitation hergeleitet werden (QED).
    (2) Der Casimir-Effekt kann auch mit Rotverschiebung (ART) hergeleitet werden.

    Siehst Du die Parallelität? Der gleiche Effekt beschrieben durch zwei Zugänge. Daher meine Annahme, der Casimir-Effekt könnte ein Zugang zur Quanten-Gravitation sein.

    Beide Herleitungen benötigen eine “Energie-Quelle” für ihre Wirkung.
    (1) (QED) Unschärfe mit m.E. fragwürdiger Energie-Bilanz (ein Teil wird ja in Arbeit umgewandelt).
    (2) (ART) Photonen im Partikel-Massefeld mit (noch) unklarer Herkunft in Platten-Partikeln (ähnlich zu Gravitonen). Auch hier wieder eine fragwürdige Energie-Bilanz, diesmal aber bezogen auf die photonenaussendenden Platten-Partikel.

    (2) ist eine “Privat”theorie – wie alles, worüber man sich zunächst Gedanken macht, bevor vielleicht mal etwas wirklich brauchbares dabei herauskommt.

    Generelle Frage zur Energie-Bilanz:

    In der QM werden Partikel durch ihre Wellenfunktion beschrieben. Dabei geht man doch immer von unendlich ausgedehnten räumlichen Wellen aus, die am wahrscheinlichsten Aufenthaltsort des Partikels im Raum ihr Maximum aufweisen.
    Die Frage ist jetzt: Wo ist die Quelle der Energie für diese Welle und wo geht diese Energie hin?

    Herzliche Grüße.

  15. #15 MartinB
    22. Juni 2012

    @SCHWAR_A
    Nur ganz knapp (hoffentlich deshalb nicht unfreundlich klingend), weil ich gerade auf dem Sprung bin…

    “Siehst Du die Parallelität? Der gleiche Effekt beschrieben durch zwei Zugänge. ”
    Ich sehe den Sinn nicht. Wenn ich aus den maxwellgleichungen die Existenz von em-Wellen herleite und dann tatsächlich em-Wellen finde, welchen Sinn soll es dann ergeben, die em-Wellen auf einem anderen Wege herzuleiten? (Es sei denn, wenn ich ame Ende die Maxwellgleichungen auf eine fundamentalere Theorie zurückführen wollte, aber das wird hier ja nicht klappen können, weil, wie gesagt, die Gravitationskonstante in die QED nicht eingeht)

    “photonenaussendenden Platten-Partikel.”
    Kein Partikel sendet hier Photonen aus, nicht mal virtuelle.

    “Wo ist die Quelle der Energie für diese Welle und wo geht diese Energie hin?”
    Verstehe ich nicht. Die Quelle der Energie ist, was imemr sie ist. Wenn ich gegen ein Elektron gegentrete, dann ist das die Quelle, wenn ich es in einem elektrischen Feld beschleunige, dann das usw.

  16. #16 SCHWAR_A
    22. Juni 2012

    @MartinB:
    “Wenn ich gegen ein Elektron gegentrete, dann ist das die Quelle, wenn ich es in einem elektrischen Feld beschleunige, dann das usw.”

    Ich kühle gedanklich ein ruhendes Partikel auf T=0: Seine Wellenfunktion bleibt erhalten. Aber es gibt keine Quelle für diese Wellen (und eigentlich auch keine Senke…), nur ein Maximimum am wahrscheinlichsten Aufenthaltsort.

    Jetzt kühle ich gedanklich die ruhenden Platten-Partikel samt Zwischenraum auf T=0: Woher kommen die (virtuellen?) Photonen?

    Herzliche Grüße.

  17. #17 Niels
    22. Juni 2012

    @SCHWAR_A
    Mal ne Frage am Rande: Warum sollte es überhaupt eine gravitative Rotverschiebung geben, wenn ein Photon von der einen Platte zur anderen fliegt?

    Geht es überhaupt darum, dass Photonen von der einen Platte zur anderen fliegen?
    Ich kann nämlich momentan überhaupt nicht nachvollziehen, wie deine Idee aussieht.
    Bitte nochmal ganz langsam und verständlich:
    Wie sorgt Rotverschiebung dafür, dass eine anziehende Kraft entsteht?

    Ist dir außerdem klar, dass der Casimir-Effekt unter bestimmten Bedingungen auch abstoßende Kräfte zwischen den Platten verursachen kann?
    Theoretisch wurde das schon lange vorhergesagt und vor kurzem auch experimentell bestätigt.
    https://www.nature.com/nature/journal/v457/n7226/abs/nature07610.html

    Ist das in deinem “Modell” auch möglich?

    Im Gravitations-Modell gibt es aber immerhin bereits Oszillatoren (½m_1·c²).

    Was? Das ist mit ziemlich größer Sicherheit Unsinn.

    ΔE = E-Quelle · gravitative Rotverschiebung · Anzahl Andockpunkte auf Quelle / Volumen(Dichte) · Volumen(Kopplung) · Anzahl Wellen zwischen Platten
    ΔE = (½m_1·c²) · 2G·(½m_1)/(c²d) · κ / (½ · 4/3 πd³) · A·d · π^4/90/(2π)

    […]
    κ Punkte auf dem Partikel
    […]

    Diese ausgesandte Energie besitzt die E-Dichte bzgl. der Halbkugel mit Radius d

    Ich hab nicht mal die leiseste Ahnung, was du da rechnest.
    Eine halbwegs richtige Formel für das ΔE der Rotverschiebung eines Photons ist allerdings schon mal nirgends zu entdecken.

  18. #18 Niels
    22. Juni 2012

    Die Betonung oben war eigentlich so geplant: “abstoßende(!) Kräfte”.

  19. #19 MartinB
    23. Juni 2012

    @SCHWAR_A
    “Aber es gibt keine Quelle für diese Wellen (und eigentlich auch keine Senke…), nur ein Maximimum am wahrscheinlichsten Aufenthaltsort.”
    So what? Ich kühle ein klassisches Punktteilchen auf Null Kelvin ab. Dann habe ich an einer Stelle eine Materiekonzentration. Aber es gibt eine Quelle für diese Konzentration.
    Eine statische Welle braucht auch keine Quelle, ne Quelle brauche ich, wenn sich was ändert, aber hier ändert sich nichts (auch im Casimir-Effekt).

    Ich gebe übrigens zu, dass ich deine Gleichungen nicht mal angeguckt habe, weil das wenig hilft, wenn man die Idee nicht versteht, die hinter den Gleichungen steckt.

  20. #20 SCHWAR_A
    24. Juni 2012

    @MartinB:
    “Eine statische Welle braucht auch keine Quelle, ne Quelle brauche ich, wenn sich was ändert”

    Statische Welle? Was ist das? Keine Welle?
    Habe ich den Begriff Wellenfunktion eines Partikels falsch verstanden?

    Die Basis von Quantenteilchen in der Wellenfunktion sind aber doch reale Wellen.
    Und Wellen brauchen mindestens eine Quelle (samt Senken), zB. das Teichen selbst im es umgebenden Feld.

    Herzliche Grüße.

  21. #21 SCHWAR_A
    24. Juni 2012

    @Niels:
    “abstoßende(!) Kräfte”

    Soweit ich weiß sind die aber zunächst bei weitem schwächer als die attraktiven Kräfte im Anziehungs-Experiment. Daher sollte man davon ausgehen, daß hierbei eine Überlagerung zweier Effekte stattfindet: der Casimir-Effekt und ein Polarisierungs-Effekt, der aufgrund ausgerichteter Dipole Abstoßung erzeugt.

    Ich habe irgendwo mal gelesen, daß das Annäherungsverhalten beim abstoßenden Experiment anders sei als 1/d^4, es geht bei extremer Annäherung eher über in Richtung 1/d², also abstoßende Ladungen. Ich finde gerade die Quelle dafür nicht…
    Kann es sein, daß das mal in Verbindung mit Teflon formuliert wurde?

    Herzliche Grüße.

  22. #22 SCHWAR_A
    24. Juni 2012

    @Niels:
    “Geht es überhaupt darum, dass Photonen von der einen Platte zur anderen fliegen?”

    Ja, bzw. Gravitonen, aber die ART-Rotverschiebung bezieht sich soweit ich weiß nur auf Photonen. Photonen im Massefeld eines aussendenden Partikels erleiden eine entsprechende Rotverschiebung, es fließt also Energie zum Sender zurück. (Gibt es Aussagen diesbezüglich über Gravitonen?)

    Gibt es genügend viele solcher Energie-Rückflüsse, in meinem Modell κ, dann ist die mögliche resultierende rückwärtsgerichtete Kraft auf andere Partikel durchaus relevant, sofern diese nicht zu weit weg sind von der Quelle.

    “Eine halbwegs richtige Formel für das ΔE der Rotverschiebung eines Photons ist allerdings schon mal nirgends zu entdecken.”

    Ich vermute, Du meinst

    z = GM/(c²d)

    mit M die Partikelmasse. Dadurch ist diese Näherung erlaubt, da d immer sehr viel größer ist als der Schwarzschild-Radius.

    Herzliche Grüße.

  23. #23 MartinB
    24. Juni 2012

    “Und Wellen brauchen mindestens eine Quelle (samt Senken), zB. das Teichen selbst im es umgebenden Feld.”
    Huh? In der Qm (standard-Interpretaion, nicht nach Bohm) ist das Teilchen die Welle. Da Teilchen keine Quelle brauchen, brauchen Wellenfunktionen auch keine. Natürlich ist jedes teilchen (und damit jede Wellenfunktion)irgendwann mal entstanden, aber das ist ja nicht, was du meinst.
    Aber die “Nullpunktfluktuationen” zwischen den Platten sind ja keine Teilchen im eigentlichen Sinne, sondern eben nichts als die Tatsache, dass es eine nicht-verschwindende Wahrscheinlichkeit gibt, einen Wert ungleich Null zu messen. Wennich zwei Platten hinstelle, dann ändert sich das Feld (oder das Wellenfunktional) zwischen den Platten nicht , und wo sich nichts ändert, brauche ich auch keine Quelle.

  24. #24 SCHWAR_A
    24. Juni 2012

    @MartinB:
    “Natürlich ist jedes teilchen (und damit jede Wellenfunktion)irgendwann mal entstanden, aber das ist ja nicht, was du meinst.”

    Das spielt aber in das Modell hinein: entstanden sind Teilchen ja aus Wellen von außen. Irgendwie müssen die aber im Teilchen stabil “gefangen” bleiben. Bleibt die Energie nicht “gefangen”, zerfällt oder zerstrahlt das Teilchen wieder.

    Hat man nicht sogar mal Experimente gemacht, die besagen, daß der Teilchen-Zerfall anders abläuft, wenn man diese mit Energie bombardiert? Dadurch erhalten die Teilchen Energie von außen und zerfallen mit geringerer Wahrscheinlichkeit.

    Herzliche Grüße.

  25. #25 Niels
    24. Juni 2012

    Eine halbwegs richtige Formel für das ΔE der Rotverschiebung eines Photons ist allerdings schon mal nirgends zu entdecken.

    Ich vermute, Du meinst
    z = GM/(c²d)
    mit M die Partikelmasse. Dadurch ist diese Näherung erlaubt, da d immer sehr viel größer ist als der Schwarzschild-Radius

    Bitte was?
    Die Formel für die gravitative Rotverschiebung in der Schwarzschild-Metrik ist https://upload.wikimedia.org/wikipedia/en/math/5/0/8/5088217333022c3068640b7737ae0bc8.png
    Für einen (unendlich) weit entfernten Empfänger/Beobachter verkürzt sich das auf

    z = {1/(Wurzel[1-Rs/R])}-1 .

    Rs = 2*G*M/c^2 ist der Schwarzschildradius, bei der radialen Entfernung R wird das Photon gesendet.

    Was machst du jetzt, damit du von einer dieser beiden Formeln auf
    z = GM/(c²d) = 1/2*Rs/d kommst?
    Was ist überhaupt d?

    Photonen im Massefeld […] erleiden eine entsprechende Rotverschiebung […] (Gibt es Aussagen diesbezüglich über Gravitonen?)

    Gravitative Rotverschiebung bei Gravitonen?
    Ich glaub, Einstein rotiert gerade mit Überlichtgeschwindigkeit im Grab. 😉

    es fließt also Energie zum Sender zurück

    Nö, so einen Vorgang gibt es dabei in der ART nicht.

    .
    Aber jetzt zurück zur Hautpsache:

    Geht es überhaupt darum, dass Photonen von der einen Platte zur anderen fliegen?

    Ja
    […]
    Photonen im Massefeld eines aussendenden Partikels erleiden eine entsprechende Rotverschiebung

    Ich glaube, ich hab deine Idee immer noch nicht verstanden.
    Stellst du dir ungefähr folgendes vor?
    Ein “Partikel” (also ein Atom?) der einen Platte sendet ein Photon aus. Dieses Photon wird durch das “Gravitationsfeld” dieses aussendenden Partikels rotverschoben. Usw.
    Richtig?

    Nun, da gibt es mehrere Probleme:

    1. Warum soll es überhaupt eine Netto-Rotverschiebung geben?
      Wenn wir uns vorstellen, dass unsere beiden Platten jeweils aus nur einem Partikel bestehen, wird das Photon vielleicht wirklich nach dem Aussenden rot-verschoben.
      Vor dem Absorbieren wird es dann aber natürlich durch das Empfänger-Artikel wieder zurück blau-verschoben.
      Davon abgesehen ist die Rotverschiebung eines Photons durch ein Atom so unfassbar winzig, dass das deine ganze Gleichung kaputt macht.
    2. Die ART ist für solche Betrachtungen bei Punktteilchen gar nicht geeignet. Das siehst du sofort, wenn du dir die obigen Formeln anschaust.
      Wo strahlt das Punkt-“Partikel” denn das Photon aus, d.h. was ist Rsource?
      Man kommt in Teufels Küche, wenn Rsource kleiner als der Schwarzschild-Radius des Partikels ist.
    3. Du tust so, als wäre das “Gravitationsfeld” des aussendenden Partikels das einzige vorhandene Gravitationsfeld.
      Das ist natürlich falsch.
      Die ART-Gleichungen sind doch nicht-lineare DGLs.
      Selbst wenn sich die beiden Platten im leeren Raum extrem weit von allen Gravitationsquellen befinden, müsste man sich richtigerweise die durch diese beiden Platten erzeugte Metrik anschauen und nicht die Metrik des einzelnen Partikels.
      Das hat dann aber mit Sicherheit nichts mehr mit Schwarzschild zu tun.
      Wenn dabei allerdings nichts anderes passiert als das ein Photon von der einen Platte zur anderen fliegt, sind wir wieder bei 1, soweit es die Rotverschiebung betrifft.
  26. #26 Niels
    24. Juni 2012

    Was machst du jetzt, damit du von einer dieser beiden Formeln auf

    z = GM/(c²d) = 1/2*Rs/d kommst?

    Ach, die Antwort ist mir gerade spontan gekommen. Ist auch wirklich ziemlich leicht.

    z = {1/(Wurzel[1-Rs/R])}-1 taylorentwickeln ergibt
    z = (1-1) +(1/2*Rs/R ) +…
    Die höheren Ordnung lässt du weg. Damit ist dann auch meine Frage beantwortet, was d ist. (Warum du erzählst, d wäre der Radius irgend einer Halbkugel, versteh ich aber nicht.)

    Sorry, hat ein bisschen geklemmt. Du hast mich verwirrt, weil du dazu etwas vom Schwarzschild-Radius erzählt hast.
    Das ist ziemlich irreführend.
    Diese Näherung ist nämlich genau das Ergebnis, dass man auch erhält, wenn man einfach und naiv mit Hilfe der newtonschen Gravitationstheorie rechnet.

  27. #27 MartinB
    25. Juni 2012

    @Niels, SCHWAR_A
    Und hinzu kommt, dass diese Vorstellung
    “Ein “Partikel” (also ein Atom?) der einen Platte sendet ein Photon aus.”
    generell nicht zum Casimir-Effekt passt, weil da keine teilchen emittiert werden, nicht mal virtuelle. (Man kann natürlich störungstheoretisch versuchen, das mit Feynman-Diagrammen zu beschreiben – auch wenn das nicht so klar ist, wie das genau gehen soll – aber in der Summe über alle Feynmandiagramme kommt ein stationärer Zustand raus, insofern ist die Vorstellung mit virtuellen teilchen auf jeden Fall irreführend.)

  28. #28 SCHWAR_A
    25. Juni 2012

    @Niels:
    “Warum soll es überhaupt eine Netto-Rotverschiebung geben?”

    Ich betrachte nicht die Wellenlänge des Vorwärts-Photons. Stattdessen benutze
    ich die Rotverschiebung, um zu beschreiben, daß Energie rückwärts zum aussendenden Oszillator fließt. (Im folgenden als vereinfachter linearer Verlauf…)
    Über eine Strecke d integriert fließt dann zum einen Oszillator E(d=0) + ΔE zurück, zum anderen (wir nehmen gleiche Frequenzen an) dasselbe, nur eben in Gegenrichtung, also mit anderem Vorzeichen.
    Die Summe beschreibt das Netto zurückfließender Energie an jedem Punkt x der Strecke d

    E_netto(x) = E(x=0) + ΔE x/d – ( E(x=0) + ΔE – ΔE x/d)
    E_netto(x) = 2ΔE x/d – ΔE

    also ist E_netto(x=d) = ΔE in Richtung des jeweils aussendenden (anderen) Oszillators.

    Herzliche Grüße.

  29. #29 SCHWAR_A
    25. Juni 2012

    @Niels:
    “…Punkt-“Partikel”…”

    Mein Modell-Partikel ist nie ein Punkt. Es hat immer eine spezifische Größe und somit einen Radius, der der Hälfte seiner Abstrahlungs-Wellenlänge entspricht, also λ=2π2r. Dadurch gibt es nie Konflikte mit “Unendlich”.
    Außerdem erhält es dadurch eine “Oberfläche” und auf ihr jede Menge Planck-Längen-Quadrate. Jedes dieser Quadrate kann in diesem Modell Quelle einer Welle sein (meine κ Andockpunkte weiter oben in der Casimir-Umformung). Daher der riesige Multiplikator, der zu einem meßbaren Effekt führt.

    Herzliche Grüße.

  30. #30 MartinB
    25. Juni 2012

    @SCHWEAR_A
    Je mehr du schreibst, desto weniger verstehe ich die Idee dahinter. Du postulierst irgendwelche ausgedehnten teilchen mit einem spezifischen radius in den Casimirplatten, die irgendwelche Photonen aussenden, deren Rotverschiebung dann Energie zurück in die Platten überträgt?

  31. #31 SCHWAR_A
    25. Juni 2012

    @MartinB:
    “Du postulierst irgendwelche ausgedehnten teilchen mit einem spezifischen radius in den Casimirplatten, die irgendwelche Photonen aussenden, deren Rotverschiebung dann Energie zurück in die Platten überträgt?”

    Ja. Dadurch wirkt in diesem Modell die dadurch zurückfließende Energie auf die Oszillatoren der gegenüberliegenden Platte dort anziehend.

    Herzliche Grüße.

  32. #32 MartinB
    25. Juni 2012

    @SCHWAR_A
    Was ist auf der “anderen Seite”, wo die Photonen absorbiert werden?
    Und warum sollen überhaupt Photonen ausgesandt werden?
    Eigentlich war der Satz von mir in der Hoffnung geschrieben, du würdest mal einmal ganz klar von A bis Z erläutern, welche Annahmen du in deinem Modell machst.
    So hat das ganze etwas von Gedankenlesen.

  33. #33 Niels
    25. Juni 2012

    @MartinB

    Und hinzu kommt, dass diese Vorstellung
    “Ein “Partikel” (also ein Atom?) der einen Platte sendet ein Photon aus.”
    generell nicht zum Casimir-Effekt passt

    Das ist mir schon klar. Mir ist auch klar, dass die Gravitation beim Casimir-Effekt keine Rolle spielen kann.
    Ich wollte nur verstehen, was es überhaupt mit der Rotverschiebungs-Idee auf sich hat.

    @SCHWAR_A

    Ich betrachte nicht die Wellenlänge des Vorwärts-Photons. Stattdessen benutze ich die Rotverschiebung, um zu beschreiben, daß Energie rückwärts zum aussendenden Oszillator fließt. […]
    Über eine Strecke d integriert fließt dann zum einen Oszillator E(d=0) + ΔE zurück, zum anderen (wir nehmen gleiche Frequenzen an) dasselbe, nur eben in Gegenrichtung, also mit anderem Vorzeichen.

    Mein Modell-Partikel ist nie ein Punkt. Es hat immer eine spezifische Größe und somit einen Radius, der der Hälfte seiner Abstrahlungs-Wellenlänge entspricht, also λ=2π2r.

    Dein “Modell” setzt offenbar ne große Menge unbegründeter und seltsamer Postulate voraus.
    Dass das ganze dann in fundamentalem Widerspruch zur ART und zur Quantenphysik/QED steht macht dir nichts aus?
    Davon abgesehen passt deine Formel nicht mal zu den Beobachtungen!
    Noch schlechter kann ein neues physikalisches Modell doch gar nicht abschneiden. 😉

    Wie sollen wir überhaupt sinnvoll über dieses “Modell” sprechen?
    Vor allem, wenn man dazu die ganze momentane Physik als falsch voraussetzen muss?
    Auf welcher Grundlage können wir dann überhaupt noch diskutieren?

  34. #34 SCHWAR_A
    25. Juni 2012

    @Niels:
    “wenn man dazu die ganze momentane Physik als falsch voraussetzen muss?”

    “Dass das ganze dann in fundamentalem Widerspruch zur ART und zur Quantenphysik/QED steht macht dir nichts aus?”

    Doch, das würde mir eine ganze Menge ausmachen. Ich sehe die Widersprüche aber anscheinend nicht.
    Eine wirkliche QG-Theorie sollte im Endeffekt vollständig in bestehende Theorien konvertierbar sein.

    “Davon abgesehen passt deine Formel nicht mal zu den Beobachtungen!”

    Welche Formel meinst Du und wo paßt sie nicht zu welchen Beobachtungen?
    Ich habe doch lediglich eine allgemein als gültig anerkannte Formel des Casimir-Effektes anders umgeformt. “Beobachtet” wird nur die Kraft zwischen den Platten.

    Herzliche Grüße.

  35. #35 MartinB
    25. Juni 2012

    @SCHWAR_A
    “Ich habe doch lediglich eine allgemein als gültig anerkannte Formel des Casimir-Effektes anders umgeformt.”
    Eben. Und genau das ist Unsinn, weil diese Formel aus der QED abgeleitet werden kann. Wenn du dieselbe Formel ganz anders herleitest, wo ist dann der Beitrag zur Casimir-Kraft, der direkt ohne QG aus der QED folgt?
    Jede QG wird die QED als Grenzfall enthalten, und in diesem Grenzfall sollten dann hoffentlich alle Effekte, die auf Gravitation beruhen verschwinden, damit wir die bekannte QED wiederbekommen.

  36. #36 SCHWAR_A
    25. Juni 2012

    @MartinB:
    “Jede QG wird die QED als Grenzfall enthalten, und in diesem Grenzfall sollten dann hoffentlich alle Effekte, die auf Gravitation beruhen verschwinden, damit wir die bekannte QED wiederbekommen.”

    Diesen Satz kann ich aber doch auch umdrehen und dadurch nichts zerstören:
    “Die Effekte der QED als Grenzfall der QG können komplett durch Gravitation erklärt werden.”
    Wie ich schon sagte, “ineinander konvertierbar”.

    “wo ist dann der Beitrag zur Casimir-Kraft, der direkt ohne QG aus der QED folgt?”

    Das ist doch kein Wettstreit “verschiedener” Dinge, sondern nur eine Uminterpretation, also kein Widerspruch, sondern eine andere Herangehensweise, eine Alternative.
    Daher meine Idee, der Casimir-Effekt birge einen Zugang zur QG in sich: einfach durch Umformen des Planckschen Wirkungsquantums zu G mit Hilfe der Planck-Masse.

    Herzliche Grüße.

  37. #37 MartinB
    25. Juni 2012

    “Die Effekte der QED als Grenzfall der QG können komplett durch Gravitation erklärt werden.”
    Das ist doch Unsinn – die Quantengravitation ist doch keine Alternative zur QED, sondern eine allgemeinere Theorie. Ich versuche es mal mit einem Beispiel (ich hoffe, es ist nicht zu schief): In der klassischen Physik kann ich das Verhalten eines Kreisels über die klassischen Gleichungen basierend auf der Drehimpulserhaltung herleiten. Es würde keinen Sinn ergeben, wenn du jetzt ankommst und stattdessen eine Herleitung findest, die die gleiche Formel über eine Wechselwirkung mit dem Higgsfeld erzeugt, denn dann musst du gleichzeitig erklären, warum die andere Erklärung nicht korrekt ist.

    Ich habe den Eindruck, du verstehst ganz fundamental nicht, was es bedeutet, wenn eine Theorie eine andere impliziert oder enthält.

    Aber so oder anders: Kannst du bitte mal ganz genau erklären, wie dein Modell funktioniert? Welche Teilchen tun was? Wie? Welche Wechselwirkungen nimmst du an? Ich habe nämlich nach wie vor nicht verstanden, wer da warum welche Photonen aussenden soll und wie die dann womit wechselwirken.

  38. #38 Niels
    25. Juni 2012

    @SCHWAR_A

    “Davon abgesehen passt deine Formel nicht mal zu den Beobachtungen!”
    Welche Formel meinst Du und wo paßt sie nicht zu welchen Beobachtungen?
    Ich habe doch lediglich eine allgemein als gültig anerkannte Formel des Casimir-Effektes anders umgeformt. “Beobachtet” wird nur die Kraft zwischen den Platten.

    Sorry, da habe ich mich wirklich unklar und viel zu kurz ausgedrückt.
    Mir ging es zum einen darum, dass man deine Formeln für die “Modell-Partikel” und für die “Energie-Rückflüsse” bei der Rotverschiebung nicht nur beim bzw. als Casimir-Effekt beobachten müsste, sondern auch bei allen anderen physikalischen Vorgängen.
    Oder kannst du alle physikalischen Prozesse mit Hilfe dieser “Modell-Partikel” und deiner “Energie-Rückflüsse” erklären?
    Zum anderen stellt sich die Frage, warum die gängigen aus der QED und der ART hergeleiteten Formeln, mit denen man alle anderen physikalischen Vorgänge (und eben auch den Casimir-Effekt) richtig beschreiben kann, hier nicht angewendet werden dürfen.
    Wenn man sie benutzen dürfte, bekäme man nämlich einen doppelt so starken Casimir-Effekt.

    Dazu hat MartinB in der Zwischenzeit aber auch schon etwas gesagt.
    Das scheinst du aber nicht verstanden zu haben. Wobei klemmt es denn genau?

    “Dass das ganze dann in fundamentalem Widerspruch zur ART und zur Quantenphysik/QED steht macht dir nichts aus?”
    Doch, das würde mir eine ganze Menge ausmachen. Ich sehe die Widersprüche aber anscheinend nicht.

    Einige dieser Widersprüche hatte ich doch direkt über diesem Satz als Zitate aufgeführt?

    Ausgedehnten Teilchen mit einem spezifischen Radius, die nur Photonen einer bestimmten Wellenlänge im Verhältnis zu diesem Radius abstrahlen könne, widersprechen der Quantenphysik.
    Das Zeug mit den Planck-Längen-Quadraten vermutlich auch. Da versteh ich aber überhaupt nicht, was du uns sagen willst, deswegen ist das nicht ganz klar.

    Rotverschiebung, die zu Energieflüssen zum Sender und in Gegenrichtung mit anderem Vorzeichen zum Empfänger führt, widerspricht der ART. Dass diese Energieflüsse auch noch Kräfte verursachen, macht es noch schlimmer.
    Dass man hier nur die Gravitation eines einzelnen Partikels betrachtet und man die Einflüsse der Platten (und der Erde) vernachlässigen können soll, ist auch Unsinn.
    In der ART geht es um die Gesamtgeometrie, die von allen Massen usw. verursacht wird.
    Es ist ausdrücklich Unsinn, sich einfach einen Teil herauszugreifen und den Rest zu ignorieren. Vor allem, wenn man als Teil nur die gravitative Wirkung von etwas betrachtet, dass man sonst Testmasse nennen würde.

  39. #39 SCHWAR_A
    26. Juni 2012

    @MartinB & Niels:
    [Martin’s aktuellsten Beitrag fühle ich irgendwie auf unsere Diskussion bezogen…;-(]

    Möglicherweise kenne ich jetzt unser Verständigungs-Problem:
    Damit mein Modell als QG (TOE?) gesehen werden kann, muß sie allgemeiner sein und daher die anderen Theorien ableitbar machen, also enthalten.

    Die ART verbiegt die Raumzeit aufgrund der verwendeten Metrik und eliminiert dadurch Kräfte.
    Die QG dagegen erklärt u.a. diese (in der ART eliminierten) Kräfte in unverbogener Raumzeit unter Berücksichtigung von Laufzeit der vermittelnden Kräfte.
    Dadurch erscheint bei gleichzeitiger Anwendung alles doppelt, was natürlich Quatsch ist. Man wendet entweder QG an, oder ART, beide müssen zum selben Ergebnis führen.

    Bei der QED kenne ich mich noch nicht so gut aus, aber soweit ich weiß, gibt es dort Kräfte, die nicht eliminiert werden.
    Die QG, eher TOE, muß diese Kräfte erklären können. Auch hier wendet man entweder QG/TOE an, oder QED, beide müssen zum gleichen Resultat führen.

    “…denn dann musst du gleichzeitig erklären, warum die andere Erklärung nicht korrekt ist.”
    Die QG/TOE besagt nicht, daß die anderen Theorien falsch sind, sondern enthält sie.

    Sind wir soweit einer Meinung?

    Herzliche Grüße.

  40. #40 MartinB
    26. Juni 2012

    @SCHWAR_A
    “Martin’s aktuellsten Beitrag fühle ich irgendwie auf unsere Diskussion bezogen”
    Nö, gar nicht. Wir reden einfach nur aneinander vorbei, das kommt in den besten Kreisen vor.

    “Die QG dagegen erklärt u.a. diese (in der ART eliminierten) Kräfte in unverbogener Raumzeit unter Berücksichtigung von Laufzeit der vermittelnden Kräfte.”
    Huh? Den Satz verstehe ich nicht wirklich. Natürlich berücksichtigt die ART auch die Laufzeit der Kräfte (oder Raumzeitkrümmungen).

    “Bei der QED kenne ich mich noch nicht so gut aus, aber soweit ich weiß, gibt es dort Kräfte, die nicht eliminiert werden.”
    Was du mit “Kräfte eliminieren” meinst, verstehe ich anscheinend auch nicht. Wenn du versuchst (oder der Meinung bist, es müsste möglich sein), alle Phänomene der QED auf Gravitation zurückzuführen, dann bist du definitiv auf dem Holzweg. (Verwechselst du QG als eine Theorie, die nur die Gravitation quantisiert, mit einer TOE?)

    “Man wendet entweder QG an, oder ART, beide müssen zum selben Ergebnis führen.”
    Ja, aber es kann nicht so sein, dass man in der QG einen Effekt hat, der zu einem Phänomen führt, und in der ART einen *anderen* Effekt, der zum selben Phänomen führt, denn dann wäre der Effekt eben doppelt so stark.

    “Die QG/TOE besagt nicht, daß die anderen Theorien falsch sind, sondern enthält sie.”
    Aber du kannst eben nicht zwei unterschiedliche und nichts miteinander zu tun habende Erklärungen für dasselbe Phänomen haben.

  41. #41 SCHWAR_A
    26. Juni 2012

    @MartinB:
    “Natürlich berücksichtigt die ART auch die Laufzeit der Kräfte (oder Raumzeitkrümmungen).”

    Klar, sie verbiegt ja durch die Metrik die Raumzeit entsprechend.
    Was ich meinte bezieht sich auf Laufzeiten bzgl. bewegter Massen, so wie hier skizziert.

    “Verwechselst du QG als eine Theorie, die nur die Gravitation quantisiert, mit einer TOE?”

    Das sieht so aus. Das habe ich inzwischen auch bemerkt.

    Im dt. Wikipedia zu “Quantengravitation” steht dazu:
    “Zusätzlich gilt die Quantengravitation als möglicher Kandidat einer TOE (Theory Of Everything), da sich mit der Vereinheitlichung von allgemeiner Relativitätstheorie und Quantentheorie vermutlich die gesamten physikalischen Eigenschaften des Universums aus einer einzigen Formel herleiten lassen, vom Hebelgesetz bis zur Zeitdilatation durch Fortbewegung nahe der Lichtgeschwindigkeit.”

    Herzliche Grüße.

  42. #42 MartinB
    26. Juni 2012

    “Was ich meinte bezieht sich auf Laufzeiten bzgl. bewegter Massen, so wie hier skizziert.”
    ??? Das ist doch in der ART ebenfalls drin?

  43. #43 Niels
    27. Juni 2012

    @SCHWAR_A

    “Natürlich berücksichtigt die ART auch die Laufzeit der Kräfte (oder Raumzeitkrümmungen).”
    Klar, sie verbiegt ja durch die Metrik die Raumzeit entsprechend.
    Was ich meinte bezieht sich auf Laufzeiten bzgl. bewegter Massen, so wie hier skizziert.

    Deine Rechnung im anderen Thread kann ich wieder überhaupt nicht nachvollziehen.
    Ich glaube da kommt wieder nur rein zufällig eine bekannte Formel heraus. (Bzw. du weißt schon vorher, was herauskommen soll, und setzt dann ziemlich willkürlich so lange Dinge gleich und ineinander ein, bis die gewünschte Formel herauskommt.)

    “Es scheint, als sei auch die ART ebenso wie die SRT „die klassische Physik“, erweitert um die Berücksichtigung von Laufzeit.”

    Nö. Das ist weder für die SRT noch für die ART richtig.
    Außerdem hast du offenbar überhaupt nicht verstanden, wo in der ART eigentlich überhaupt Laufzeiten auftreten.
    Wenn sich in der ART zwei Massen mit konstanter Geschwindigkeit zueinander bewegen, fühlt jede Masse eine (Schein)Kraft, die auf die instantane Position des anderen Körpers zeigt.
    Da gibt es keine Verzögerung durch irgendwelche Laufzeiten.
    Die physikalischen Gesetze müssen in allen Bezugssystemen gleich sein und man kann ja in ein System wechseln, in dem der eine Körper ruht.
    Ein statisches Felder bzw. eine statische Raumzeitgeometrie muss sich doch gar nicht ausbreiten. Das ist etwas, dass einfach da ist.
    Deswegen passt es eigentlich nicht, von einer Laufzeit der Gravitation zu sprechen.
    (Sorry, dass ich hier geometrische Vorstellung und Feldvorstellung vermische. Sich rein in der geometrischen Vorstellung auszudrücken ist extrem mühsam.)

    Die Periheldrehung des Merkur kommt absolut nicht irgendwie durch “den Einfluss der Laufzeit der Gravitationswirkung auf die Planetenbahn” zustande, wie du im anderen Thread behauptest.
    Dieses Phänomen lässt sich rein mit Hilfe der Schwarzschild-Metrik beschreiben, also mit Hilfe des Feldes einer homogenen, nicht geladenen und nicht rotierenden Kugel.
    Das Feld ist also wieder völlig statisch, so etwas wie “Laufzeiten der Gravitationswirkung” oder generell irgend etwas, dass sich ausbreitet, gibt es deswegen überhaupt nicht.

    Auch die gravitative Rotverschiebung hat anders als dort von dir behauptet nicht das geringste mit irgendwelchen endlichen Laufzeiten zu tun.
    (Na gut, vielleicht mit Laufzeiten von Uhren. :-))

  44. #44 SCHWAR_A
    27. Juni 2012

    @Niels & MartinB:
    In der ART ist Raum und Zeit derart in der Metrik enthalten, daß man mit instantanem Feld rechnen kann. Das ist sehr praktisch.

    Meine Ideen spielen sich im 3-dimensionalen Euklidischen Raum statt, also ohne spezielle Metrik.

    Sorry, daß das nicht sofort klar wird. Ich dachte, das sieht man sofort…

    Im Euklidischen Raum müssen m.E. Kräfte und Laufzeiten berücksichtigt werden. Das macht die Mathematik zwar schwieriger, muß aber letztlich zum selben Ergebnis führen wie bei der Verwendung der praktischen Feldgleichungen.

    “…du weißt schon vorher, was herauskommen soll, und setzt dann ziemlich willkürlich so lange Dinge gleich und ineinander ein, bis die gewünschte Formel herauskommt.”

    So nicht! Ich habe mich mit Binären Sternsystemen beschäftigt, und versucht, die verzögerte Wirkung der Positionsänderungen aufeinander zu verstehen (wieder im Euklidischen Raum). Dabei ist mir der Zusammenhang aufgefallen. Ich wußte vorher nicht, daß die Formel der gravitativen Rotverschiebung damit irgendetwas zu tun haben könnte. Es ist aber faszinierend, daß sie sich quasi nebenbei ergibt…

    Herzliche Grüße.

  45. #45 MartinB
    27. Juni 2012

    @SCHWAR_A
    Nur damit keine Missverständnisse aufkommen: Man kann die ART auch komplett über Feldgleichungen im euklidischen Raum umformulieren, soweit ich es sehe – siehe das Buch von Weinberg.

  46. #46 SCHWAR_A
    27. Juni 2012

    @MartinB:
    “Man kann die ART auch komplett über Feldgleichungen im euklidischen Raum umformulieren, soweit ich es sehe – siehe das Buch von Weinberg. ”

    Den Weinberg hab’ ich leider nicht zur Verfügung.

    Das ist genau das, was ich meine:
    egal welche Metrik man zugrundelegt, es muß im Endeffekt exakt das selbe herauskommen, nur, daß es möglicherweise in der einen Metrik einfacher zu rechnen geht als in der anderen. Deswegen sind die anderen Rechnungen aber nicht falsch, sie sind evt. nur umständlicher.
    Allerdings decken andere Rechnungen manchmal zusätzliche oder andere Interpretationen auf, die in der einfachen Rechenmethode einfach implizit enthalten sind, ohne direkt offensichtlich zu werden.

    Wie geht denn Weinberg mit der Laufzeit der Gravitation im Euklidischen Raum um?
    Er müßte für ein Binäres System gleicher Massen zum selben Ergebnis komme, wie ich…

    Eine etwas andere Frage (bzw. 2) zur ART:
    Ein Elektron, das in einem Massefeld seiner Geodäte folgt:
    – Erfährt dieses Elektron eine Beschleunigung?
    – Sendet dieses Elektron dann Synchrotronstrahlung aus?

    Herzliche Grüße.

  47. #47 MartinB
    27. Juni 2012

    “Wie geht denn Weinberg mit der Laufzeit der Gravitation im Euklidischen Raum um?”
    Die Gravitation läuft mit Lichtgeschwindigkeit, wie sonst? Ob das irgendwo explizit gerechnet wird, kann ich nicht sagen.

    Die Frage mit dem Elektron ist ein bisschen trickreich. Ich zitiere mal Wikipedia
    “Dies bedeutet insbesondere, dass jeder Körper sich unabhängig von seiner Masse in einem Schwerefeld (bei Abwesenheit anderer Kräfte) gleich bewegt. (Geladene Körper sind davon aufgrund der Synchrotronstrahlung ausgeschlossen.)”

  48. #48 SCHWAR_A
    27. Juni 2012

    @MartinB:
    “Geladene Körper sind davon aufgrund der Synchrotronstrahlung ausgeschlossen.”

    Heißt das jetzt, man weiß es nicht genau, oder eher, daß man hier einen QED-Effekt (noch) nicht mit der ART vereinigen kann, obwohl man ihn mißt?

    Der Satz enthält ja eigentlich nicht wirklich eine Begründung:
    “aufgrund der Synchrotronstrahlung” tut ja so, als ob Synchrotronstrahlung die ART beeinflusse, obwohl es durch die ART gar keine Beschleunigung, sprich Synchrotronstrahlung gibt. Da beißt sich was gegenseitig.

    Was wird denn darüber an der Uni gelehrt?

    Herzliche Grüße.

  49. #49 MartinB
    27. Juni 2012

    @SCHWAR_A
    Ich habe über das Problem irgendwo mal was gelesen, kann mich aber im Moment nicht erinnern, wie genau das Argument funktioniert. Soweit ich mich entsinne, ergibt sich da bei korrekter Behandlung keine Widersprüche – beschleunigte Ladungen verlieren Energie, und dann gilt die Aussage der ART natürlich so nicht mehr, genau so, wie sie nicht mehr gilt, wenn das teilchen auf Grund von Reibung oder sonst etwas Energie verliert.
    Hier gibt’s dazu ne ausführliche Rechnung:
    https://books.google.de/books?id=ks78dHJ6cw8C&pg=PA87&lpg=PA87&dq=%22general+relativity%22+synchrotron+radiation&source=bl&ots=qWQvLqtn6r&sig=yPVd6ie6CigmzCug5FGZemdQMDs&hl=de&sa=X&ei=wPbqT6usPMzhtQaR-OnLBQ&ved=0CHgQ6AEwBw#v=onepage&q=%22general%20relativity%22%20synchrotron%20radiation&f=false

    Du könntest die Frage mal auf physicsforums stellen,d a bekommst du vermutlich schnell ne Antwort von jemandem, der tiefer drinsteckt als ich.

  50. #50 SCHWAR_A
    27. Juni 2012

    @MartinB:
    …auf Seite 92 wechselt er dann aber doch zur klassischen Kreis-Bewegung und baut wieder Laufzeit ein, wodurch er dann den Energieverlust erhält…

  51. #51 MartinB
    27. Juni 2012

    @SCHWAR_A
    ich hab’s mir nicht im Detail angeguckt, es war mehr ein versteckter Hinweis, doch mal selbst zu suchen… 😉

  52. #52 Niels
    27. Juni 2012

    @MartinB
    Ich kenn das so:
    Nach dem Äquivalenzprinzip sind Gravitationsfelder und Beschleunigung lokal nicht voneinander zu unterscheiden. Ein frei fallender Beobachter spürt selbst weder Kraft noch Beschleunigung, obwohl ihn das Gravitationsfeld bzw. die Raumkrümmung “beschleunigt”.
    Das Äquivalenzprinzip gilt allerdings nur lokal, also nur für extrem kleine raumzeitliche Abstände.
    Da das elektrische Feld eines geladenen Teilchens nicht nur lokal vorliegt, sondern prinzipiell sogar den ganze Raum füllt, ist das Äquivalenzprinzip auf geladene Teilchen nicht anwendbar.
    Man kann das wohl auch so beschreiben, dass ein geladenes Teilchen im Gravitationsfeld irgendwie mit seinem eigenen elektrischen Feld wechselwirkt.
    Deswegen bewegen sich Ladungen nicht auf Geodäten und deswegen strahlen sie im Gravitationsfeld (beispielsweise ein Elektron, das um die Erde kreist), während sie nicht strahlen, wenn sie im Gravitationsfeld festgehalten werden (wie beispielsweise Objekte ruhende Objekte auf der Erde).

    So ganz klar ist mir das ganze aber noch nicht.
    Ein paar Paper dazu:
    https://arxiv.org/abs/grqc/0306052
    https://arxiv.org/abs/0806.0464

  53. #54 MartinB
    27. Juni 2012

    @Niels
    Danke für die Erklärung und die Links. Wenn ich nachher Lust habe, frage/suche ich mal auf physicsforum, ich bin mir ziemlich sicher, dass sich da ein passender Experte äußern kann.

  54. #55 MartinB
    27. Juni 2012

    Erwartungsgemäß wurde das schon diskutiert:
    https://www.physicsforums.com/showthread.php?t=576478&highlight=synchrotron
    Erwartungsgemäß ist die Antwort nicht einfach…
    Siehe auch
    https://www.physicsforums.com/showthread.php?t=160533
    mit der Quelle
    https://xxx.lanl.gov/abs/gr-qc/0006037

  55. #56 SCHWAR_A
    27. Juni 2012

    @MartinB & Niels:
    …viel zu lesen…
    Vielen Dank.
    Vielleicht können wir das ja bald fortsetzen…

    Herzliche Grüße.

  56. #57 Niels
    28. Juni 2012

    @MartinB
    Es gibt erstaunlich viele aktuelle Veröffentlichungen, die sich aber teilweise sogar darin widersprechen, ob Strahlung ausgesendet wird oder nicht.
    Siehe zum Beispiel meinen vierten Link für eine Beispielveröffentlichung der Gegenposition.
    Wie die verschiedenen Erklärungsansätze der Mehrheitsposition zusammen passen kann ich im Moment leider nicht nachvollziehen.
    Das steht mit auf meiner Liste der Dinge, die ich mir mal genauer anschauen möchte, wenn ich richtig viel Zeit übrig habe.

    Es gibt ein ganz aktuelles 380 Seiten schweres Buch nur über dieses Thema, in dem die verschiedenen Paper und Erklärungen besprochen und “gewertet” werden.
    https://www.amazon.com/Uniformly-Accelerating-Charged-Particles-Equivalence/dp/3540684697

    Mal aus reiner Neugier gefragt:
    Noch deinem momentanen Stand, wie würdest du die Sache erklären?

  57. #58 MartinB
    28. Juni 2012

    @Niels
    Nach meinem momentanen Stand habe ich auf Anhieb keine Idee – ich finde deinen Erklärungsansatz, dass ein geladenes teilchen immer als ausgedehnt zu betrachten ist und deswegen das Äquivalenzprinzip nicht gilt, sehr einleuchtend, aber da es anscheinend andere Ansichten gibt, halte ich mich an mein Lieblings-Zitat aus Star Trek:
    “Captain, the most elementary and valuable statement in science, the beginning of wisdom, is, “I do not know”. I do not know what that is, sir. “

  58. #59 Niels
    28. Juni 2012

    @MartinB
    Sehr schönes Zitat. Danke, werde ich mir merken.

    Das Problem bei der Sache mit dem Äquivalenzprinzip ist aber zum Beispiel, dass man als Gedankenexperiment auch eine Ladung in einem unendlich ausgedehnten homogenen Gravitationsfeld betrachten kann.
    Da hätte man keine Probleme mit der lokalen Gültigkeit.

    Das erwähnte Buch kann man übrigens auch über die Uni einsehen:
    https://www.springerlink.com/content/978-3-540-68469-5/#section=215906&page=1
    (Oder auf anderem Wege im Internet als Ebook “besorgen”.)
    Ich hab gerade das Schlusskapitel “Conclusion” gelesen. Klarer wurde mir dadurch aber auch nichts.
    Der Autor ist aber übrigens absolut davon überzeugt, dass das Äquivalenzprinzip hier selbstverständlich doch anwendbar ist.

  59. #60 SCHWAR_A
    1. Juli 2012

    @Niels:
    “Deswegen bewegen sich Ladungen nicht auf Geodäten und deswegen strahlen sie im Gravitationsfeld (beispielsweise ein Elektron, das um die Erde kreist), während sie nicht strahlen, wenn sie im Gravitationsfeld festgehalten werden (wie beispielsweise Objekte ruhende Objekte auf der Erde).
    So ganz klar ist mir das ganze aber noch nicht.”

    Ich habe ein schönes Gedankenexperiment dazu entdeckt:

    Wir stellen uns eine Box vor, die sich entlang einer Geodäte bewegt.
    In dieser Box (ausgedehnt, also nicht streng lokal) seien ein geladenes Teilchen und ein ungeladenes, beide in Ruhe.

    Keines der beiden Teilchen wird seinen Platz relativ zum anderen und relativ zur Box verändern. Das Äquivalenzprinzip ist erfüllt.

    Insbesondere wird das geladene Teilchen nicht (synchrotron-)strahlen, zumindest nicht im Box-Frame.
    Strahlt es jedoch in einem anderen Frame, dann strahlt auch das ungeladene Teilchen darin.

    Müßte dieses Ergebnis nicht als Fakt für alle weiteren Interpretationen erfüllt sein?

    Herzliche Grüße.

  60. #61 MartinB
    1. Juli 2012

    @SCHWAR_A
    “Keines der beiden Teilchen wird seinen Platz relativ zum anderen und relativ zur Box verändern. Das Äquivalenzprinzip ist erfüllt.”
    Wenn sich beide entlang einer Geodäte in einem nicht-konstanten Schwerefeld bewegen, dann werden sie sich doch relativ zueinander bewegen wegen der Gezeiteneffekte?

  61. #62 SCHWAR_A
    1. Juli 2012

    @MartinB:
    …meinst Du damit, daß es 2 Teilchen prinzipiell unmöglich ist, der gleichen Geodäten zu folgen, weil sie nie am selben Ort zur selben Zeit sein können?
    Demzufolge auch nicht die Teilchen der Box?

    Im Gedankenexperiment sei das Schwerefeld jetzt für die gesamte Box konstant.
    Wie sieht es dann mit der Synchrotron-Strahlung je nach Beobacher-Frame aus?

    Herzliche Grüße.

  62. #63 MartinB
    1. Juli 2012

    @SCHWAR_A
    Zu Punkt 1, ja, außer im homogenen Schwerefeld.

    Punkt 2 war ja genau das Problem, das Niels oben angesprochen hat.
    https://www.scienceblogs.de/hier-wohnen-drachen/2012/06/qft-fur-alle-wir-verstehen-alles.php#comment339497

  63. #64 SCHWAR_A
    1. Juli 2012

    @MartinB:
    “Punkt 2 war ja genau das Problem, das Niels oben angesprochen hat.”

    …mit einer kleinen Erweiterung, um die es mir jetzt ging:
    Ich betrachte ein geladenes und ein ungeladenes Teilchen gleichzeitig.

    Worauf ich hinaus will ist, daß es m.E. egal sein muß, ob das Teilchen geladen ist oder nicht. Aus einem anderen Frame gesehen müssen entweder beide strahlen, oder keines.
    Das habe ich so aber noch nie gehört. Leider kann man keine ungeladenen Teilchen “kreis”-beschleunigen, um das zu überprüfen…

    Herzliche Grüße.

  64. #65 MartinB
    1. Juli 2012

    1. Ein ungeladenes teilchen strahlt mit sicherheit nicht, wie sollte es?

    2. Nimm ne dünne Schnur und knote sie an ein schweres Teilchen und wirst sehen, dass es kein Problem ist, ein ungeladenes Teilchen im Kreis zu bewegen. (Ob hinreichend schnell, ist eine andere Frage)

  65. #66 SCHWAR_A
    1. Juli 2012

    @MartinB:
    Wiki zu Bremsstrahlung:
    “The moving particle loses kinetic energy [durch Abbremsen], which is converted into a photon because energy is conserved.”

    Müßte das nicht für jedes Teilchen gelten, auch ungeladene?

    Herzliche Grüße.

  66. #67 MartinB
    1. Juli 2012

    @SCHWAR_A
    Nein, ungeladene Teilchen haben keine Möglichkeit, Photonen zu erzeugen.
    Bremsstrahlung kann man sich ein bisschen als die Selbstwechselwirkung eines Teilchens mit seinem eigenen E-Feld vorstellen.
    Wenn du es genau wissen willst (und wissen willst, wo die Schwierigkeiten liegen), lies umbedingt die Feynman Lectures, vol II, ich glaube so um Kap 28 rum.

  67. #68 SCHWAR_A
    1. Juli 2012

    @MartinB:
    “lies umbedingt die Feynman Lectures, vol II, ich glaube so um Kap 28 rum.”

    Danke, werd’ ich machen…

    Herzliche Grüße.

  68. #69 SCHWAR_A
    2. Juli 2012

    @MartinB:
    …im Prinzip redet Feynman im Kapitel 28 aber auch davon, daß es sowas wie eine EM-Masse von Teilchen (allgemein, nicht nur die geladenen) geben muß, man nur noch keine Theorie dazu hat, die zufriedenstellend ist. [in Kapitel 28.5]
    “[in Kapitel 28.3: “Und es besteht die faszinierende Möglchkeit, daß der mechanische Teil überhaupt nicht vorhanden ist – daß die Masse gänzlich elektromagnetisch ist.”]

    Wenn also auch ein Neutron EM-Masse besitzt, verhält es sich bzgl. Selbstwechselwirkung bei Beschleunigung ähnlich zum Elektron: es will seine Energie konstant halten und strahlt daher die für die Richtungsänderung aufgenommene Energie wieder ab.

    Herzliche Grüße.

  69. #70 MartinB
    3. Juli 2012

    @SCHWAR_A
    Ein Neutron ist ja auch aus geladenen Teilchen zusammengesetzt und hat ein magnetisches Moment; als beispiel für ein Teilchen ohne em-Wechselwirkung ist es deshalb ungeeignet.

  70. #71 SCHWAR_A
    3. Juli 2012

    @MartinB:
    Welches Teilchen wäre denn geeignet? Gibt es überhaupt Teilchen mit Ladung 0 UND OHNE magnetisches Moment?

  71. #72 MartinB
    3. Juli 2012

    @SCHWAR_A
    Ja, Neutrinos.
    Falls es sie gibt, auch die WIMPs.

  72. #73 SCHWAR_A
    3. Juli 2012

    @MartinB:
    Was gilt denn für Photonen? Auch Ladung 0 und KEIN magn.Moment?

  73. #74 SCHWAR_A
    3. Juli 2012

    @MartinB & Niels:
    hier habe ich noch eine Überlegung:

    In einem binären Sternsystem sind beide Massen (quasi) ungeladen.
    Ihr Abstand verringert sich.
    Das wird als Quelle für Gravitationswellen angesehen.
    Das ist doch nichts anderes als eine Abstrahlung der Energie, bzw. des Drehimpulses, die ja beide im System erhalten bleiben müssen, oder?

    Stellt man sich jetzt zwei einzelne neutrale Teilchen vor, die sich umkreisen, so müßte das Verhalten gleich bleiben: auch diese müssen abstrahlen (solange sie sich tatsächlich auch annähern können, ohne durch Quantenbedingungen daran gehindert zu werden).

    Erweitert stellt man sich dann zwei neutrale zusammengesetzte Atome vor, die sich gegenseitig umkreisen. Auch hier müßte sich ihr Abstand verringern, also Abstrahlung erfolgen.

    Könnte man daraus ableiten, daß Gravitationswellen eigentlich nur speziell polarisierte (quadrupolige) EM-Wellen sind, allerdings seeehr langwellige?

    Herzliche Grüße.

  74. #75 Niels
    3. Juli 2012

    “Keines der beiden Teilchen wird seinen Platz relativ zum anderen und relativ zur Box verändern. Das Äquivalenzprinzip ist erfüllt.”

    Wenn sich beide entlang einer Geodäte in einem nicht-konstanten Schwerefeld bewegen, dann werden sie sich doch relativ zueinander bewegen wegen der Gezeiteneffekte?

    Das stimmt natürlich. Deswegen steht auch in einigen Büchern, der Begriff Gravitationsfeld habe in der ART gar keine echte physikalische Bedeutung mehr, sondern die Physik stecke im Gezeitenfeld.
    Die Christoffelsymbole, die den Gravitationsfeldstärken enstsprechen, lassen sich ja wegtransformieren. Der Krümmungstensor, der dem Gezeitenfeld zugeordnet ist, dagegen nicht.

    Ich würde auf die Frage aber so antworten:
    Geodäten sind doch nur im flachen Raum Parallelen. Gekrümmte Räume lassen sich doch sogar dadurch definieren, dass sich der Abstand zwischen benachbarten Geodäten ändert.
    Vielleicht wirds dadurch ein bisschen klarer.

    Übrigens war die Sache für Feynman ganz einfach (Feynman Lectures On Gravitation):
    Ladungen im Gravitationsfeld strahlen nicht. Im Gravitationsfeld festgehaltene Ladungen dagegen schon.
    Man müsse nur das Äquivalenzprinzip ernst nehmen, dann ergebe sich das automatisch und ohne tieferes Nachdenken. 😉

    @SCHWAR_A

    In einem binären Sternsystem sind beide Massen (quasi) ungeladen.
    Ihr Abstand verringert sich.
    Das wird als Quelle für Gravitationswellen angesehen.

    Nein. Bei einem binären Sonnensystem weist die Massenverteilung ein Quadrupolmoment auf.
    Deswegen werden Gravitationswellen abgestrahlt.
    Die Abstandsverringerung ist nicht die Ursache für die Abstrahlung sondern vielmehr eine Folge davon.

    Das ist doch nichts anderes als eine Abstrahlung der Energie, bzw. des Drehimpulses, die ja beide im System erhalten bleiben müssen, oder?

    Gravitationswellen tragen Energie, Impuls und Drehimpuls, das stimmt.

    Energieerhaltung ist in der ART aber ein schwieriges Thema, damit argumentiert man besser nicht. Ist hier aber auch gar nicht nötig.

    Stellt man sich jetzt zwei einzelne neutrale Teilchen vor, die sich umkreisen, so müßte das Verhalten gleich bleiben

    Richtig.

    Könnte man daraus ableiten, daß Gravitationswellen eigentlich nur speziell polarisierte (quadrupolige) EM-Wellen sind, allerdings seeehr langwellige?

    Nein.

    Gavitationswellen werden gerne mit EM-Wellen verglichen.
    Das liegt daran, dass die linearisierten Einsteinschen Feldgleichungen auf eine Wellengleichung führen, die der Wellengleichung in der Elektrodynamik sehr ähnlich sieht.
    Beide Wellen breiten sich mit Lichtgeschwindigkeit aus und werden ähnlich erzeugt,
    EM-Wellen nämlich durch beschleunigte Ladungen und Gravitationswellen durch beschleunigte Massen.
    Allerdings ist die Wellengleichung in der ART eine Gleichung für die Metrik.
    Gravitationswellen sind also Störungen der Raumzeit selbst, die sich als Welle ausbreiten. Sie besitzen auch zwei Polarisationen, nicht nur eine.
    Gravitationswellenpulse können sogar auch kollabieren und ein Schwarze Löcher erzeugen.

  75. #76 SCHWAR_A
    3. Juli 2012

    @Niels:
    “Die Abstandsverringerung ist nicht die Ursache für die Abstrahlung sondern vielmehr eine Folge davon.”

    “Wellen breiten sich mit Lichtgeschwindigkeit aus, … Gravitationswellen durch beschleunigte Massen.”

    Die Abstandsverringerung im Binärsystem ist doch Teil der Gesamt-Beschleunigung: ein Teil führt zur elliptischen bzw. kreisförmigen Bahn, der andere führt zur Abstandsverringerung, wegen der räumlichen Oszillation des tatsächlichen Baryzentrums durch die zeitliche Verzögerung der Gravitation bzgl. der sich konträr bewegenden Massen (im euklidischen Raum!). Letzteren Teil kann man natürlich implizit verstecken, wenn man eine geeignete nicht-euklidische Metrik verwendet, z.B. wie in der ART.

    Daher würde ich hierbei nicht Ursache und Wirkung vertauschen.

    Diese Baryzentrums-Oszillation um das Ruhe-Baryzentrum herum (im euklidischen Raum!) ist es doch, die das Quadrupol-Moment generiert: je nach Blickrichtung auf diese Oszillation weisen die abgestrahlten Wellen unterschiedliche Polarisationen auf, von kreisförmig über elliptisch bis zu linear.

    Herzliche Grüße.

  76. #77 MartinB
    3. Juli 2012

    @Niels
    Danke für den Tipp mit den Feynman Lectures on Gravitation.

  77. #78 Niels
    3. Juli 2012

    @MartinB
    Feynman möchte die ART aus der Quantenphysik heraus entwickeln.
    Das ist also ein total unüblicher Ansatz und die Vorlesung war speziell an Physiker gerichtet, die schon die QED kannten.
    Dazu zeigt Feynman erst einmal, dass das entsprechende Teilchen ein masseloses Spin2 Teilchen sein muss. Dann berechnet er wie in der QED Feynman-Graphen und Amplituden.
    Dann macht er Störungsrechnung und schafft es schließlich durch trickreiche Berücksichtigung der Beobachtungen (Periheldrehung, klassischer Grenzfall,..) irgendwie zur Einstein-Hilbert-Wirkung zu kommen.
    Eine Motivation dafür, dass Feynman diese Vorlesung überhaupt gehalten hat, war wohl, dass er sich erhoffte, auf diesem Wege hin zu einer Quantengravitation zu kommen.
    Dieser Teil der Vorlesung ist im Buch allerdings weggelassen, weil er eben nirgends hin geführt hat.

    Erst in der zweiten Hälfte schaut er sich die ART mit Hilfe der üblichen Differentialgeometrie an.

    Ich hab mir das Buch nur sehr oberflächlich angeschaut, aber für den ersten Teil verstehe ich viel zu wenig von Quantenfeldtheorien, um ihn beurteilen zu können.
    Der zweite Teil wird mittlerweile meiner Meinung nach anderswo aber deutlich besser, klarer und ausführlicher behandelt.
    Ist aber auch kein Wunder. Das Buch hat ja auch nur 220 Seiten und die Vorlesung wurde schon 1962-1963 gehalten.

    Kurz gesagt hab ich also keine Ahnung, ob das Buch aufrund des ersten Teiles eine Empfehlung wert ist oder nicht.

    @SCHWAR_A
    Da hab ich leider wieder unzulässig ART-Sprache und klassische-Physik-Sprache vermischt.
    Wenn sich zwei Massen völlig ohne weitere Einflüsse umkreisen, fallen diese beiden Massen natürlich frei.
    In der ART-Sicht gibt es hier also gar keine Beschleunigung. Die Gravitationswellen werden nur deswegen abgestrahlt, weil die Massenverteilung ein Quadrupolmoment hat und das nun mal den Formeln zu Folge die Quelle von Gravitationsstrahlung ist.
    Zumindest fällt mir da momentan keine anschaulichere Beschreibung ein.

    Zu deiner Idee mit der “zeitliche Verzögerung der Gravitation” als Ursache hab ich oben schon mal etwas geschrieben, als es um den anderen Thread ging.
    Ich halte diese Sichtweise wie gesagt für falsch. Da kann ich mich aber natürlich auch irren.

    Vielleicht kannst du dazu noch etwas sagen, MartinB?

  78. #79 Niels
    3. Juli 2012

    Noch was am Rande:
    Seit dem mittlerweile gelösten Kommentarproblem bei Scienceblogs zeigt die Kommentarliste bei HwD bei mir immer Unsinn an.
    Momentan zum Beispiel:

    Kommentare

    SCHWAR_A · 18.06.12 · 14:12 Uhr
    QFT für alle: Wir verstehen Alles

    Wissenschaft und Schreie · 25.06.12 · 19:30 Uhr
    Das kleine Troll-Handbuch

    BreitSide · 01.07.12 · 13:00 Uhr
    Auch Säugetierknochen brauchen mal ne Pause…

    Chris · 24.06.11 · 21:53 Uhr
    Das kleinste Schmiedeteil der Welt

    Auf der Hauptseite und bei den anderen Blogs funktioniert es aber (anscheinend) richtig.
    Liegt das Problem an meinem Ende?

  79. #80 MartinB
    3. Juli 2012

    @Niels
    Kleines Missverständnis: ich meinte, danke für’s Erinnern an die Feynman Lectures – die waren es, die mich letztes Jahr dazu gebracht haben, mich wieder mit QFT zu beschäftigen, weil Feynman darin so viele Dinge in der QFT quasi aus dem Handgelenk vorzaubert. Ich finde gerade den ersten Teil extrem lesenswert, habe aber noch nicht alles wirklich gründlich angeguckt (bald sind zum Glück Ferien).

    “In der ART-Sicht gibt es hier also gar keine Beschleunigung.”
    Wirklich nicht? Lokal ist natürlich (in einer geschlossenen Kiste o.ä.) keine Beschleunigung feststellbar, aber das im System beider Massen insgesamt Beschleunigung stattfindet, ist doch schon klar, oder nicht?

    “Ich halte diese Sichtweise wie gesagt für falsch.”
    Ich auch, habe ich auch oben schon gesagt.

    Was die Kommentare angeht, das ist auf allen Blogs so: Es wird immer der erste Kommentar eines Strangs angezeigt, nicht der letzte.
    Angeblich sollen angesichts der Probleme unsere Techniker aber an einem zügigenUmzug auf WordPress arbeiten (zügig heißt vermutlich: Kommt schon in 100 Jahren, nicht in 1000).

  80. #81 Niels
    3. Juli 2012

    @MartinB

    Wirklich nicht? Lokal ist natürlich (in einer geschlossenen Kiste o.ä.) keine Beschleunigung feststellbar, aber das im System beider Massen insgesamt Beschleunigung stattfindet, ist doch schon klar, oder nicht?

    Da hab ich mich schlecht ausgedrückt.
    Wenn man die Massen betrachtet, fallen sie frei, oder?
    Wenn man in der Elektrodynamik sagt, dass beschleunigte Ladungen EM-Wellen abstrahlen, meint man also etwas anderes, als wenn man in der ART sagt, dass beschleunigte Massen Gravitationswellen abstrahlen.
    Hm, das krieg ich irgendwie nicht anständig formuliert. Ist klar, was ich meine?
    Oder red ich grad Stuss?

  81. #82 Niels
    3. Juli 2012

    “Ich halte diese Sichtweise wie gesagt für falsch.”
    Ich auch, habe ich auch oben schon gesagt.

    Sorry, hatte ich schon wieder vergessen.

    Das Prinzip hinter den Kommentar-Anzeigen hab ich nicht durchschaut.
    Aber wenn man daran arbeitet wird ja hoffentlich bald alles gut.
    Gibts wenigstens ein paar neue Funktionen nach dem Umzug auf WordPress?

  82. #83 MartinB
    4. Juli 2012

    “Hm, das krieg ich irgendwie nicht anständig formuliert. Ist klar, was ich meine? ”
    Ich glaube schon. Man sollte wohl mal einen Satelliten mit ein paar Coulomb aufladen…

    Wobei ich mich im Moment frage, ob nicht die Astrophysiker für das Problem zumindest eine pragmatische Lösung haben – die Synchrotronstrahlung müsste doch massive Auswirkungen auf Jets usw. haben. Oder wird das durch die immer vorhandenen Magnetfelder dominiert?

    “Gibts wenigstens ein paar neue Funktionen nach dem Umzug auf WordPress?”
    Wie heißt es so schön: die Hoffnung stirbt zuletzt. Aber sie stirbt.

    Angeblich sollte dann sogar direktes LaTeX für Formeln gehen; ich glaub’s wenn ich’s sehe.

  83. #84 SCHWAR_A
    4. Juli 2012

    @Niels:
    “Zumindest fällt mir da momentan keine anschaulichere Beschreibung ein.”

    Das Quadrupolmoment in der ART ist doch eine dynamische Komponente, oder?
    Ich meine damit, daß es dabei doch um eine dynamische Ortsänderung der Masse geht, quasi um das sich zyklisch verlagernde Baryzentrum eines Systems.

    Ist das nicht anschaulich genug?

    Herzliche Grüße.

  84. #85 Niels
    4. Juli 2012

    @MartinB

    Oder wird das durch die immer vorhandenen Magnetfelder dominiert

    Genau so hab ich das mit den Jets zumindest verstanden.
    Hab mich aber auch schon gewundert, warum sich das nicht durch irgendwelche astronomische Beobachtungen klären lässt.

    “Hm, das krieg ich irgendwie nicht anständig formuliert. Ist klar, was ich meine? ”

    Ich glaube schon.

    Kannst du es vielleicht besser artikulieren? Hilft mir vielleicht beim Verständnis.

    @SCHWAR_A

    Ich meine damit, daß es dabei doch um eine dynamische Ortsänderung der Masse geht, quasi um das sich zyklisch verlagernde Baryzentrum eines Systems.

    Was ist denn eine “dynamische Ortsänderung”?
    Warum soll sich das Baryzentrum “zyklisch verlagern”? Wenn sich zwei gleich große Massen umkreisen liegt es doch immer genau in der Mitte?

  85. #86 SCHWAR_A
    4. Juli 2012

    @MartinB:
    “genau in der Mitte”

    …aber nur im “statischen” Fall. Sobald sich jedoch beide Massen umkreisen, “sieht” eine der Massen die jeweils andere auf einer “früheren” Position (im euklidischen Raum!).
    Damit ist das “wirksame” Baryzentrum etwas näher als das der “Mitte”, es hat eine etwas andere Richtung, und außerdem ist es immer in Bewegung, um das “mittlere” Baryzentrum herum – es hinkt immer der tatsächlichen Richtung zur gegenüber liegenden Masse hinterher. Beide Massen sehen jeweils “ihr” eigenes “wirksames” Baryzentrum.

    Für eine Testmasse außerhalb des Durchmessers des Binärsystems variiert das auf sie bezogene Baryzentrum ebenfalls (im euklidischen Raum!) und sie detektiert diese Variation als eine Störung der Raumzeit, eine Gravitationsfeld-Schwingung.

    Herzliche Grüße.

  86. #87 MartinB
    4. Juli 2012

    @Niels
    “Kannst du es vielleicht besser artikulieren? Hilft mir vielleicht beim Verständnis. ”
    Im Moment nicht. Ich müsste erst nochmal ein bisschen lesen, um wieder reinzukommen, ist aber im Moment schlecht, weil ich abends diese Woche gar keine Zeit habe…

  87. #88 MartinB
    29. November 2019

    @alle
    Aus irgendeinem Grund wird dieser Artikel mit Spam-Kommentaren überflutet.
    Ich habe deshalb hier die Kommentare deaktiviert. Falls Ihr einen Kommentar habt, hinterlasst ihn bei einem anderen Teil der Serie oder schreibt mir eine mail.
    MartinB